Quantcast
  • Register
PhysicsOverflow is a next-generation academic platform for physicists and astronomers, including a community peer review system and a postgraduate-level discussion forum analogous to MathOverflow.

Welcome to PhysicsOverflow! PhysicsOverflow is an open platform for community peer review and graduate-level Physics discussion.

Please help promote PhysicsOverflow ads elsewhere if you like it.

News

PO is now at the Physics Department of Bielefeld University!

New printer friendly PO pages!

Migration to Bielefeld University was successful!

Please vote for this year's PhysicsOverflow ads!

Please do help out in categorising submissions. Submit a paper to PhysicsOverflow!

... see more

Tools for paper authors

Submit paper
Claim Paper Authorship

Tools for SE users

Search User
Reclaim SE Account
Request Account Merger
Nativise imported posts
Claim post (deleted users)
Import SE post

Users whose questions have been imported from Physics Stack Exchange, Theoretical Physics Stack Exchange, or any other Stack Exchange site are kindly requested to reclaim their account and not to register as a new user.

Public \(\beta\) tools

Report a bug with a feature
Request a new functionality
404 page design
Send feedback

Attributions

(propose a free ad)

Site Statistics

205 submissions , 163 unreviewed
5,047 questions , 2,200 unanswered
5,345 answers , 22,709 comments
1,470 users with positive rep
816 active unimported users
More ...

  Semileptonic decays of the $B_c$ meson

+ 6 like - 0 dislike
1513 views

I am struggling with calculating the exclusive semileptonic $B_c^+\rightarrow J/\psi l^+\nu_l$ decay. I learnt that the amplitude is given by a product of the leptonic current $L^{\mu}$ and the hadronic current $H^{\mu}$ $$ \mathcal{M}(B_c\rightarrow J/\psi l^+\nu_l)=\frac{G_F}{\sqrt{2}}V_{cb}L^{\mu}H_{\mu} $$ where $V_{cb}$ is the CKM parameter, $L^{\mu}$ and $H^{\mu}$ are expressed as $$ L^{\mu}=\bar{u}_l\gamma^{\mu}(1-\gamma^5)v_{\nu},\quad H^{\mu}=\langle J/\psi|J^{\mu}(0)|B_c\rangle $$ where $J^{\mu}$ is the $V$-$A$ weak current. However, I did not know how this result can be derived. Could anyone provide some help?

There is a second problem. On the tree-level, we have the following Feynman diagram enter image description here

If we calculate $\bar{b}\rightarrow\bar{c}l^+\nu_l$ as a three-body decay in the electroweak theory (not the four-fermion approximation adopted above), how does it relate to $B_c^+\rightarrow J/\psi l^+\nu_l$?

This post imported from StackExchange Physics at 2014-06-15 16:45 (UCT), posted by SE-user soliton
asked Jun 12, 2014 in Theoretical Physics by soliton (110 points) [ no revision ]
Maybe it will be interesting to take a look at this question: physics.stackexchange.com/questions/116499/…

This post imported from StackExchange Physics at 2014-06-15 16:45 (UCT), posted by SE-user Melquíades
Very nice! It is really helpful.

This post imported from StackExchange Physics at 2014-06-15 16:45 (UCT), posted by SE-user soliton

1 Answer

+ 1 like - 0 dislike

For this process, the interaction Hamiltonian is given by:

$$\mathcal{H}_{\rm int}=-\frac{g}{\sqrt 2}\left(V_{cb}\bar{b}_L\gamma^\mu c_L W^-_\mu+\bar{\nu}_L\gamma^\mu\ell_L W^+_\mu\right).$$

After integrating-out the heavy bosons, we obtain the following Hamiltonian

$$\mathcal{H}_{\rm eff}=-\dfrac{G_F}{\sqrt{2}}V_{cb}[\bar{b}\gamma^\mu(1-\gamma_5)c][\bar{\nu}\gamma^\mu(1-\gamma_5)\ell],$$ where $G_F/\sqrt{2}=g^2/(8 m_W^2)$ is the Fermi constant.

To obtain the tree-level amplitude for the process $B_c\to J/\psi \ell^+ \nu$, we consider the following matrix element

$$\mathcal{A}(B_c\to J/\psi \ell^+ \nu)=-i\langle J/\psi \,\ell^+\, \nu_\ell |\mathcal{H}_{\rm eff} | B_c\rangle. $$ If you write explicitly the leptonic fields in terms of creation and annihilation operators, then you will notice that $$\mathcal{A}(B_c\to J/\psi \ell^+ \nu)=i \dfrac{G_F}{\sqrt{2}}V_{cb}\bar{u}_\nu \gamma^\mu (1-\gamma_5) v_\ell \langle J/\psi| \bar{b}\gamma^\mu(1-\gamma_5)c| B_c\rangle.$$

Note that we have isolated the hadronix matrix element from the rest. Now, if we are able to find this element by using Lattice QCD methods or experimental results, then we will be able to compute the decay rate and other observables. [However, I don't think this is possible for this particular transition at present.]

For your second question, if you consider only valence quarks in the mesons, then you are using a tree-level approximation to describe hadronic states. This is a crude approximation, because QCD is non-perturbative at low energies. You can improve it by computing high-order QCD corrections, but you will never have a reliable result.

This post imported from StackExchange Physics at 2014-06-15 16:45 (UCT), posted by SE-user Melquíades
answered Jun 14, 2014 by Melquíades (40 points) [ no revision ]
Thank you very much! Is it possible to introduce something like parton distribution function to relate the tree-level approximation to the decay of $B_c$ meson?

This post imported from StackExchange Physics at 2014-06-15 16:45 (UCT), posted by SE-user soliton
Usually we use Lorentz and parity symmetry to express the hadronic matrix element in terms of functions called form factors. These functions can be fitted from experimental data, if we have experimental access to the differential branching ratio, or it can be obtained by Lattice QCD numerical simulations.

This post imported from StackExchange Physics at 2014-06-15 16:45 (UCT), posted by SE-user Melquíades
However, this is the very complicated for the problem you mentioned. Since the final meson is a vector particle, you can show that you need several of these functions, because of the rich spin structure. One can find a much simpler framework in the transitions $B_c\to \eta_c$, because in this case the final particle is a pseudoscalar meson and you need only two form-factors (in the Standard Model).

This post imported from StackExchange Physics at 2014-06-15 16:45 (UCT), posted by SE-user Melquíades
Maybe you can introduce parton distribution functions, but I believe that we don't have direct experimental access to these quantities. Remember: these particles are very unstable and we cannot do scattering experiment as we do with protons.

This post imported from StackExchange Physics at 2014-06-15 16:45 (UCT), posted by SE-user Melquíades
Thank you again for your comments. It helps a lot.

This post imported from StackExchange Physics at 2014-06-15 16:45 (UCT), posted by SE-user soliton
You are welcome! Please, accept the answer if you find it useful. =)

This post imported from StackExchange Physics at 2014-06-15 16:45 (UCT), posted by SE-user Melquíades

Your answer

Please use answers only to (at least partly) answer questions. To comment, discuss, or ask for clarification, leave a comment instead.
To mask links under text, please type your text, highlight it, and click the "link" button. You can then enter your link URL.
Please consult the FAQ for as to how to format your post.
This is the answer box; if you want to write a comment instead, please use the 'add comment' button.
Live preview (may slow down editor)   Preview
Your name to display (optional):
Privacy: Your email address will only be used for sending these notifications.
Anti-spam verification:
If you are a human please identify the position of the character covered by the symbol $\varnothing$ in the following word:
p$\hbar$ys$\varnothing$csOverflow
Then drag the red bullet below over the corresponding character of our banner. When you drop it there, the bullet changes to green (on slow internet connections after a few seconds).
Please complete the anti-spam verification




user contributions licensed under cc by-sa 3.0 with attribution required

Your rights
...